LSAT and Law School Admissions Forum

Get expert LSAT preparation and law school admissions advice from PowerScore Test Preparation.

 Administrator
PowerScore Staff
  • PowerScore Staff
  • Posts: 8917
  • Joined: Feb 02, 2011
|
#32601
Complete Question Explanation

Must Be True—CE. The correct answer choice is (E)

This stimulus contains a fact set discussing medical malpractice suits against doctors and how they could be avoided. It turns out that, in most cases, the patients’ perceptions that their doctors have acted negligently or carelessly is what prompts the patients to file malpractice suits. And perhaps the patients have good reason. The professor tells us that many doctors are now less compassionate than in the past because they view medicine as a science rather than as an art.

Also, the doctors face economic incentives that encourage them to treat patients rudely. The doctors not only treat patients rudely, but also patronize them and discourage them from asking questions. The professor states that lawsuits could be avoided if doctors learned to listen better to patients.

Not surprisingly, given that the stimulus is a fact set, this is a Must Be True question. While some Must Be True stimuli permit strong prephrases based on an inference that is apparent from combining the facts, this is not one of those occasions. Move on to the answer choices keeping in mind that the correct answer choice may be a restatement of any of the facts contained in the stimulus. Since the stimulus did not produce a strong prephrase, focus your attention on eliminating incorrect answer choices using the Fact Test, in which you make sure that the information in the answer choice can be proven by reference to the stimulus.

Answer choice (A): The first sentence in the stimulus stated that “most malpractice suits arise out of patients’ perceptions that their doctors are acting negligently or carelessly.” While the economic incentives to treat patients rudely may contribute to the patients’ perception of negligence or carelessness by the doctor, the facts do not support the stronger statement that those economic incentives are the main cause of the suits.

Answer choice (B): The stimulus told us that the economic incentives encourage doctors to treat patients rudely, not to view medicine as a science rather than as an art. In fact, it may be that because the doctors increasingly view medicine as a science, they are more likely to succumb to the economic incentives to treat the patients, their scientific subjects, rudely.

Answer choice (C): We are not given sufficient information to determine whether a malpractice suit is justified. Although the professor said that the suits are mostly driven by the patients’ perceptions, we have no reason to suspect that their perceptions are incorrect.

Answer choice (D): While the professor appears to consider the shift in the number of doctors viewing medicine as a science rather than an art to be problematic, it is not necessarily the case that the scientific outlook should be replaced entirely by a different approach.

Answer choice (E): This is the correct answer choice. The stimulus told us that the doctors who view medicine as a science are less compassionate toward their patients. The economic incentives influence doctors to treat their patients rudely, and “harried” doctors patronize their patients or discourage them from asking questions. Taken together, these facts support the statement that doctors’ actions create the perception that they do not care about their patients.
 nlittle
  • Posts: 18
  • Joined: Sep 09, 2017
|
#41715
Since this is a MBT question type, I was wondering how E must be true. Particularly the meaning of the word doctors in the answer choice. When I read doctors, I think "All doctors", so to me this is saying "All doctors foster, by their actions, the perception that they do not really care about their patients." I felt this was too strong of a statement and ruled it out.

The supporter listed above shows that some doctors foster this perception. When an author names a group without specifying Some/Most/All as done in this answer choice ("Doctors"), does this imply "Some Doctors"?

Could my mistake be that this question type is "most strongly supported", as the name suggests, since you seem to have only compiled support or should this truly be handled in the same fashion as a MBT (fully supported, as in all doctors foster the perception and no room for exceptions based on the fact set)?

Thank you in advance,

Nick
 James Finch
PowerScore Staff
  • PowerScore Staff
  • Posts: 943
  • Joined: Sep 06, 2017
|
#41798
Hi Nick,

The LSAT makers use "most strongly supported" or similar phrases whenever any ambiguities or vagaries exist. When referring to an entire group, we can't say for certain whether the characteristic applies to all or only some members of the group, unless such a distinction is made.

When no distinction is made within a group, be they doctors, janitors, dogs etc., we can safely assume for the purpose of the question that whatever is said about that group is true of all members of the group. So in deciding between answer choice, use the logic in the stimulus and apply it to the same scope as given in the stimulus, in this case doctors. As neither the stimulus nor the answer choices here designate any subgroups, simply apply the logic in the stimulus to doctors generally, as we take what is said in the stimulus to be factually true for the sake of the question.

Let me know if this clears things up.
 Pragmatism
  • Posts: 68
  • Joined: Jan 11, 2018
|
#43747
So, I chose the right answer choice. However, I was left with two contenders, B & E. I discredited answer choice B for introducing "the economic incentives." I perceived that phrase to be much wider in scope than what the stimulus presented, which was, "certain economic incentives." After scrutinizing the stimulus, I realized, that the author implies "all doctors" once, to possibly address those doctors that are faced with "certain economic incentives," but mainly directs the stimulus with quantifiers that separate the doctors in its entirety with those that "regard medicine as a science rather than an art, and are less compassionate as a result." However, I chose E because, while the author of the passage narrows the this stimulus to many doctors, that are faced with most malpractice due to certain economic incentives that promulgate doctors to act less compassionate as a result, the author implies all patients that encounter such mannerism by the doctor to be negligent or careless. Finally, "if doctors learned to listen better to patients, then lawsuit could be avoided," coupled with the notion that triggers most malpractice suits, gave credence to answer choice E. I want to know, does that methodology has any merit to it?
 Adam Tyson
PowerScore Staff
  • PowerScore Staff
  • Posts: 5153
  • Joined: Apr 14, 2011
|
#43756
In short, yes! Well done, pragmatism.

Here's why B should be tossed out, and quickly: the author made gave no explanation for why doctors now treat medicine as a science rather than as an art. We know there are economic incentives to treat patients rudely, and that the "science, not art" view makes them less compassionate, but there's no clear connection between the two. Maybe the "science, not art" view is a result of the way med schools teach their students? Perhaps that and the economic incentives are working together as causes of the rude treatment of patients, but the two have no causal relationship between them?

Once we eliminate the losers, whatever is left is the winner. That's the best reason for picking E here, in my opinion, but your methodology looks sound.
User avatar
 gingerale
  • Posts: 25
  • Joined: Feb 15, 2021
|
#84659
Is the causal component of this argument significant? I chose to diagram it and look for the answer that was consistent with the cause and effect chain that I created. Is this an efficient way to attack the question?
 Adam Tyson
PowerScore Staff
  • PowerScore Staff
  • Posts: 5153
  • Joined: Apr 14, 2011
|
#84678
The causal elements of this stimulus are very important, gingerale! In fact, they are the key to the whole thing! "Arise out of" and "as a result" and "incentives encourage" are all indications of cause and effect. If you can create a causal chain here, and avoid tying in the one claim that doesn't connect to the others, you will have a great prephrase that will match the correct answer.
User avatar
 Snomen
  • Posts: 35
  • Joined: Sep 30, 2021
|
#97573
On-time I selected E.., but on BR I had doubts cuz I thought that it was kinda too powerful. "Doctors foster" Sound like "all doctors". Any advice or clarification?

Thanks !
 Rachael Wilkenfeld
PowerScore Staff
  • PowerScore Staff
  • Posts: 1358
  • Joined: Dec 15, 2011
|
#97919
Hi Snomen,

"Foster" isn't a term that is particularly strong. A general definition for the term is that it encourages or promotes. It's fair to say based on the stimulus that doctors encourage the perception that they don't care about their patients by acting rudely or rushing. Since the stimulus tells us that "many" doctors behave this way, it's fair to say that doctors' behavior in general encourages the idea that doctors do not care about their patients.

Hope that helps!

Get the most out of your LSAT Prep Plus subscription.

Analyze and track your performance with our Testing and Analytics Package.